0 of 24 Questions completed
Questions:
You have already completed the quiz before. Hence you can not start it again.
Quiz is loading…
You must sign in or sign up to start the quiz.
You must first complete the following:
0 of 24 Questions answered correctly
Your time:
Time has elapsed
You have reached 0 of 0 point(s), (0)
Earned Point(s): 0 of 0, (0)
0 Essay(s) Pending (Possible Point(s): 0)
Average score |
|
Your score |
|
Which of the following is the most likely cause for the gross pathology shown in the image below?
A 36-year-old female with bilateral lower extremity weakness, hyperreflexia, and difficulty emptying her bladder gets an urgent MRI with contrast of her thoracic spine (T2 sagittal (left) and axial (right)). Which of the following is the most likely diagnosis?
The combination of lamotrigine and which of the following drugs increases the risk of lamotrigine toxicity?
An 18-year-old boy had an MRI performed after experiencing a first-time seizure shown below. What genetic disorder could be the cause of the abnormalities appreciated on imaging?
A four-year-old boy with no significant past medical history is brought to your clinic due to difficulty walking. His mother states that he began walking at 12 months of age, but in the past few months, he seems to be more fatigued and weak when walking. On physical exam, his calves seem to be abnormally large. This disorder is caused by what mutation?
Identify the lesion shown in the imaging below:
A 37-year-old male with the chief complaint of insidious unilateral hearing loss was found to have an acoustic neuroma on MRI. If a brainstem auditory evoked potential (BAEP) was performed which of the following would be most likely found?
A 64-year-old man with idiopathic Parkinson’s disease being treated with carbidopa-levodopa is experiencing significant wearing-off despite increasing frequency and dose of levodopa-carbidopa, which is now producing significant peak dose dyskinesia. Which of the following would be a reasonable next agent to prescribe to reduce this wearing-off effect?
Which of the following comorbidities would prohibit the use of dalfampridine?
A 42-year-old woman presents to the clinic with a 2-month history of distal symmetric hand weakness that started after hospitalization for severe abdominal pain. Before this hospitalization, she was drinking alcohol with her friends when she acutely developed severe abdominal pain, nausea, restlessness, altered mental status, and hallucinations. She was noted to have dark urine at the time of admission. Since then, her GI symptoms and mentation have returned to baseline, but her weakness has persisted. On exam, she has 3/5 strength of the intrinsic muscles of the hand bilaterally. Conduction velocities of the bilateral upper extremities were unremarkable. Needle EMG revealed fibrillations and decreased recruitment of motor unit action potentials in muscles innervated by the radial and ulnar nerves. Which of the following is the most likely cause of the patient’s symptoms?
Identify the lesion shown in the image below.
An MRI brain is ordered for a 6-month-old male with failure to thrive, dystonia, and regression of developmental milestones. The T2 axial images are shown below. Which of the following is most likely the diagnosis?
A 53-year-old man has had increasing difficulty falling and staying asleep over the past 3 months. This is accompanied by hallucinations, delirium, and general clumsiness. He was an orphan as a child and does not know his family’s medical history. If a sleep study is performed, during which stage of sleep would you most likely expect to find an absence of normal sleep architecture?
A 42-year-old man with a longstanding history of chronic alcohol abuse was found down unresponsive in a public park. He was intubated in the field and brought emergently to the emergency room where his blood alcohol level was 0.32%. A chest X-ray revealed aspiration pneumonia. He was admitted to the ICU, sedated, started on broad-spectrum antibiotics, and aggressively fluid resuscitated. On the second day of the hospitalization, the sedative was weaned but the patient remained comatose. An MRI was performed to identify a possible cause of his symptoms. What is the most likely diagnosis?
A 38-year-old man with a prolonged history of intravenous drug use, HIV, and poor compliance with HAART therapy is brought to the hospital by police after being seen wandering across a busy street intersection. His last CD4 count was 180 cells/microliter 8 months ago. Vitals appear within normal limits on arrival. Neurological examination shows inattention, poor short-term memory, and difficulties with abstract thought. An MRI is performed and shown below. What is the likely etiology of his symptoms?
A patient who was admitted to the neurology intensive care unit for myasthenic crisis developed ventilatory-associated pneumonia on the fifth day of his hospitalization. On the day following the initiation of antibiotic therapy, there was a significant decline in the patient’s respiratory status. Which of the following antibiotics most likely contributed to the patient’s respiratory distress?
A chronic alcoholic is admitted to the hospital for altered mental status. An MRI is performed and shown below. What symptoms/signs are most likely seen on examination?
Which way are the patient’s eyes moving at the point of the asterisk marked below?
Based on the MRI imaging shown below, which of the following structural abnormalities is likely also present?
Which of the following is the most common cause of focal clonic seizures in full-term neonates?
Which of the following is seen on muscle biopsy in patients with steroid myopathy?
A 15-year-old male comes to the clinic with his parents as they complain he has periods of the year where he will sleep up to 20 hours a day for multiple days in a row, as well as uninhibited eating and inappropriate sexual behavior. These episodes last for up to around 2 weeks at most, and then he is behaviorally back to normal in between episodes. Which is the most likely diagnosis?
Which of the following would not constitute or be concerning for a HIPAA violation?
A 60-year-old female with a past medical history of chronic alcohol use was admitted to the hospital 4 days ago for community-acquired pneumonia. Over the last 48 hours, the patient developed severe encephalopathy and generalized weakness. An axial T2 FLAIR MRI is shown. Which of the following is the most likely diagnosis?